• Shuffle
    Toggle On
    Toggle Off
  • Alphabetize
    Toggle On
    Toggle Off
  • Front First
    Toggle On
    Toggle Off
  • Both Sides
    Toggle On
    Toggle Off
  • Read
    Toggle On
    Toggle Off
Reading...
Front

Card Range To Study

through

image

Play button

image

Play button

image

Progress

1/140

Click to flip

Use LEFT and RIGHT arrow keys to navigate between flashcards;

Use UP and DOWN arrow keys to flip the card;

H to show hint;

A reads text to speech;

140 Cards in this Set

  • Front
  • Back
"All of the following describe benefits provided by ERISA covered plans, EXCEPT:
A. Retirement benefits based on age and/or service
B. Disability retirement benefits
C. Deferral of income past the end of employment
D. Medical benefits
E. Benefits primarily for nonresident aliens in plans maintained outside the U.S.
" "The correct answer is E.
This is false because plans maintained outside the U.S. primarily for nonresident aliens are not covered by ERISA.
"
"All of the following statements regarding characteristics of defined benefit plans are TRUE, EXCEPT:
A. A specified benefit is promised to be paid at retirement.
B. The employer assumes the investment risk.
C. The benefits are usually expressed as a dollar amount to be paid as an annuity.
D. Younger employees require higher contributions than older employees given the same benefit level.
E. The PBGC guarantees a portion of the benefit for certain defined benefit plans.
" "The correct answer is D.
This statement is false because defined benefit plans do not require higher contributions for younger employees given the same benefit level. They generally require higher contributions for older employees.
"
"All of the following describe types of benefits provided by welfare benefit plans, EXCEPT:
A. Scholarship funds
B. Vacation benefits
C. Retirement benefits
D. Prepaid legal services
E. Hospital care
" "The correct answer is C.
This is false because a plan that provides retirement benefits is not a welfare benefit plan.
"
"All of the following are benefits provided by a pension benefit plan, EXCEPT:
A. Retirement income to employees
B. Unemployment benefits
C. Disability retirement benefits
D. Retirement savings through individual accounts
E. Deferral of income
" "The correct answer is B.
This is false because a plan that provides unemployment benefits is not considered a pension benefit plan.
"
"All of the following statements regarding defined contribution plans are TRUE, EXCEPT:
A. A separate account is maintained for each participant.
B. The participant bears the investment risk.
C. The participant’s total benefit equals the accumulated value of the participant’s account at retirement.
D. Investment results are not guaranteed.
E. Older employees benefit more than younger employees.
" "The correct answer is E.
This statement is false because defined contribution plans do not necessarily benefit older employees more than younger employees. Contributions for younger employees benefit from the accumulated interest compounded over a longer period of time than older employees.
"
"Which of the following statements regarding employee benefit plans is/are TRUE?
I. Pension benefit plans provide retirement income.
II. Welfare benefit plans provide a variety of benefits.
III. Employee benefit plans often are provided as an integral part of a total compensation package.
A. I only
B. II only
C. I and III only
D. II and III only
E. I, II and III
" "The correct answer is E.
All statements are true.
"
"All of the following may affect the cost of benefits in a defined benefit plan, EXCEPT:
A. Contribution formula
B. Age of the employees
C. Years worked for the plan sponsor
D. Compensation of the employees
E. Investment gains or losses on plan assets
" "The correct answer is A.
This is false because the contribution formula is used in a defined contribution. A benefit formula is used to determine the cost in a defined benefit plan.
"
"Which of the following statements regarding defined contribution and defined benefit plans is/are TRUE?
I. Both types promise to pay a specific benefit at normal retirement age.
II. The participant bears the risk for the investment results for both types.
III. ERISA defines both types of plans as pension benefit plans that provide retirement benefits.
A. I only
B. III only
C. I and II only
D. II and III only
E. I, II and III
" "The correct answer is B.
Statement I is false because defined contribution plans do not provide a specified benefit at normal retirement. The benefit is based on the accumulated value of the account balance at normal retirement. Statement II is false because the participant does not bear the investment risk in a defined benefit plan. The participant does bear the investment risk in a defined contribution plan.
Statement III is the only true statement.
"
"All of the following statements regarding defined contribution plans are TRUE, EXCEPT:
A. Investment results do not affect the employer’s cost.
B. There is no promised benefit to be paid to the participant.
C. The employer’s defined contributions are the extent of the employer’s commitment to the participant’s retirement income.
D. The benefit to be paid is generally expressed as an annuity.
E. Generally, employees understand a defined contribution plan.
" "The correct answer is D.
This statement is false because defined benefit plans generally express the benefit to be paid as an annuity, not defined contribution plans.
"
"All of the following are employee benefit plans, EXCEPT:
A. Welfare benefit plans
B. Pension benefit plans
C. Plans maintained solely to comply with worker’s compensation laws
D. Plans covered by ERISA
E. Qualified plans
" "The correct answer is C.
This is false because plans maintained solely to comply with worker’s compensation laws are not covered by ERISA and are therefore not employee benefit plans.
"
"All of the following entities may sponsor a retirement plan, EXCEPT:
A. Trusts
B. Local governments
C. Corporations
D. Sole proprietorships
E. LLPs
" "The correct answer is A. §2.03 [A]
This is false because a trust is not an entity that may sponsor a retirement plan.
"
"All of the following are automatically parties-in-interest, EXCEPT:
A. A named beneficiary for a plan participant
B. A spouse of the 100% owner of the plan sponsor
C. Counsel to the plan
D. An officer of the employer organization
E. A person providing services to the plan
" "The correct answer is A.
This is false because a named beneficiary for a plan participant is not necessarily a party-in-interest.
"
"All of the following are duties of the plan administrator, EXCEPT:
A. Hire plan service providers such as the attorney
B. Rule on claims for benefits
C. Determine eligibility for plan participation
D. Draft the articles of incorporation
E. Direct distribution of benefits
" "The correct answer is D.
This is false because drafting the articles of incorporation is not a duty of the plan administrator.
"
"All of the following are considered fiduciaries, EXCEPT:
A. A paid investment consultant who renders advice regarding plan funds
B. A trustee who has discretionary authority to sell assets
C. A spouse of a participant
D. A plan administrator who is named in the plan document
E. A plan sponsor
" "The correct answer is C.
This is false because a spouse of a participant is not necessarily a fiduciary.
"
"All of the following statements regarding plan service providers are TRUE, EXCEPT:
A. The ERPA may represent the client on issues related to the entity’s tax filing.
B. The enrolled actuary may determine the minimum and maximum contribution for a defined benefit plan.
C. The investment consultant may provide investment advice to plan participants.
D. The attorney may interpret the retirement plan provisions of the IRC.
E. The accountant may prepare audited financial statements.
" "The correct answer is A.
This statement is false because the ERPA may not represent the entity before IRS on the entity’s tax issues but it may represent the plan on its Form 5500 filing.
"
"Which of the following statements regarding parent-subsidiary controlled groups is/are TRUE?
I. There are two or more organizations that have a service relationship and an ownership relationship.
II. It may include more than one subsidiary if the first subsidiary owns 80 percent or more of one or more separate employers.
III. It exists when one business (called the parent) owns at least 80% of one or more other businesses (the subsidiaries).
A. I only
B. III only
C. I and II only
D. II and III only
E. I, II and III
" "The correct answer is D.
Statement I is false because there is no requirement for a controlled group to provide services to a member of the controlled group. The service requirement applies to affiliated service groups. Statements II and III are true.
"
"All of the following are PTs, EXCEPT:
A. The sale of stock from an officer of the plan sponsor to the plan
B. A loan from the plan to the plan’s counsel
C. The purchase of land by a director of the plan sponsor from the plan
D. A purchase by the plan of a mutual fund from a brokerage firm that has no authority or control of plan investments
E. A loan from the plan to the son, who is not a participant, of the President who is a 100% owner of the plan sponsor
" "The correct answer is D.
This is false because the brokerage firm is not a party-in-interest and therefore there is no PT. The brokerage firm is selling a product, not providing services to the plan. The brokerage firm is not a fiduciary because it does not have discretionary authority over the assets of the plan nor is it giving investment advice.
"
"All of the following family members attribute their stock ownership under the attribution rules for controlled groups, EXCEPT:
A. Minor child to parent
B. Adult child to parent where the parent owns 25% of the business
C. Grandchild to grandparent where grandparent owns 75% of the business
D. Parent to adult child where child owns 75% of the business
E. Grandparent to grandchild where grandchild owns 75% of the business
" "The correct answer is B.
This is false because an adult child’s stock is attributed to the parent only if the parent owns greater than 50% of the business.
"
"Which of the following statements regarding brother-sister controlled groups is/are TRUE?
I. There must be 5 or fewer common owners.
II. The ownership percentage must satisfy an 80% common ownership test.
III. The ownership percentage must satisfy a 50% identical ownership test.
A. I only
B. III only
C. I and II only
D. II and III only
E. I, II and III
" "The correct answer is E.
All statements are true.
"
"Which of the following statements regarding employees is/are TRUE?
I. An independent contractor is considered an employee of the plan sponsor for qualified plan purposes.
II. An individual employed by the plan sponsor and considered a common law employee is an employee for qualified plan purposes.
III. A leased employee not covered by a safe harbor plan may be considered an employee of the recipient if certain conditions are met.
A. I only
B. III only
C. I and II only
D. II and III only
E. I, II and III
" "The correct answer is D.
Statements II and III are true.
Statement I is false because an individual who is an independent contractor is not considered an employee of the firm contracting for services and therefore is not considered by that firm for qualified plan purposes.
"
"All of the following statements regarding elective contributions are TRUE, EXCEPT:
A. Pre-tax elective contributions reduce the employee’s current taxable income for federal income tax purposes.
B. Pre-tax elective contributions reduce compensation for Social Security taxes.
C. Pre-tax catch-up contributions are pre-tax elective contributions in excess of an applicable limit.
D. The IRC §402(g) dollar limit is adjusted annually for cost-of-living increases.
E. The maximum elective deferral limit is $17,500 for the 2014 calendar year.
" "The correct answer is B.
This statement is false because pre-tax elective contributions do not reduce compensation for Social Security taxes.
"
"All of the following statements regarding pre-tax catch-up contributions are TRUE, EXCEPT:
A. They are additional contributions that may only be made by certain older plan participants.
B. They are classified as pre-tax catch-up contributions when the catch-up eligible participant has reached a limit in the law or the plan.
C. Individuals must attain age 50 by the date they make the contribution.
D. There is a maximum limit that is indexed annually for cost-of-living increases.
E. They have been allowed in recognition that many older people have not accumulated sufficient amounts for retirement.
" "The correct answer is C.
This statement is false because individuals expected to attain age 50 in a calendar year are catch-up eligible for that year.
"
"All of the following statements regarding 401(k) plans are TRUE, EXCEPT:
A. Designated Roth contributions are elective contributions.
B. A catch-up eligible participant may defer $23,000 in calendar year 2014 if no other limits are affected.
C. An employer may make matching contributions to a 401(k) plan.
D. An employee can defer the maximum IRC §402(g) dollar limit in each plan of two different employers in the same calendar year.
E. An employee born December 15, 1965 is eligible to make catch-up contributions beginning January 1, 2015.
" "The correct answer is D.
This statement is false because an employee can only defer the maximum IRC §402(g) dollar limit in one calendar year regardless of the number of plans.
"
"All of the following statements regarding characteristics of profit sharing plans are TRUE, EXCEPT:
A. They must contain a specific contribution allocation formula.
B. They may be used to reward employees for their efforts.
C. The employer may exercise discretion over the amount contributed to the plan each year.
D. Contributions must be based on actual profits.
E. The maximum amount the employer may deduct on its tax return is 25% of total eligible compensation of all participants.
" "The correct answer is D.
This statement is false because contributions do not have to be based on actual profits.
"
"All of the following statements regarding characteristics of an ESOP are TRUE, EXCEPT:
A. It may be used to create a market for privately held employer stock.
B. It generally invests more than 50% of its assets in employer stock.
C. It may borrow money to purchase employer stock.
D. It may be used as a method to transfer ownership of a company to its employees.
E. It may be in the form of a defined benefit plan.
" "The correct answer is D.
This statement is false because contributions do not have to be based on actual profits.
"
"All of the following statements regarding defined contribution plans are TRUE, EXCEPT:
A. A money purchase plan obligates the employer to contribute a specific amount or percentage of the participant’s compensation to the plan each year.
B. A KSOP is a combination 401(k) and ESOP plan.
C. A stock bonus plan is a profit sharing plan that generally distributes shares of company stock to terminated participants.
D. A money purchase plan contribution formula generally will not exceed 25% of compensation.
E. A 401(k) plan requires employees to make mandatory contributions.
" "The correct answer is E.
This statement is false because 401(k) plans do not require employee contributions.
"
"Which of the following statements regarding IRAs is/are TRUE?
I. The traditional IRA allows certain individuals to contribute the lesser of $5,500 for 2014 or 100% of compensation.
II. The employer will set up the traditional IRA and the Roth IRA for the employees.
III. A Roth IRA allows an individual to contribute nondeductible contributions to the IRA.
A. I only
B. II only
C. I and III only
D. II and III only
E. I, II and III
" "The correct answer is C.
Statement II is false because the individual will set up the traditional IRA and the Roth IRA not the employer.
"
"All of the following statements regarding 403(b) plans are TRUE, EXCEPT:
A. Certain tax-exempt organization may sponsor a 403(b) plan.
B. They may be funded through the use of mutual funds held in a custodial account.
C. The sections in the IRC that govern 403(b) plans are the same as those that govern all other qualified plans.
D. Originally, these plans had to be invested in annuity contracts.
E. 403(b) plans were called tax-sheltered annuities.
" "The correct answer is C.
This statement is false because IRC §401(a) governs other qualified plans, not §403(b).
"
"All of the following statements regarding 457 plans are TRUE, EXCEPT:
A. They are a qualified plans.
B. They do not need to comply with top-heavy rules.
C. They may be sponsored by a State.
D. They may be sponsored by a tax-exempt organization.
E. The employees may elect to defer compensation.
" "The correct answer is A.
This statement is false because 457 plans are not qualified plans. They are nonqualified deferred compensation plans.
"
"All of the following statements regarding ESOPs are TRUE, EXCEPT:
A. It may deduct dividends paid on shares contributed to the ESOP.
B. Investment of plan assets in company stock automatically creates an ESOP.
C. It may generate cash flow for the employer to finance its growth.
D. The purchased shares represent the collateral on the ESOP loan if the ESOP has borrowed money to purchase company stock.
E. The employer’s cash contributions are used as loan repayments if the ESOP has borrowed money to purchase company stock.
" "The correct answer is B.
This statement is false because investment of plan assets in company stock does not automatically create an ESOP. Other plan types may invest up to 10% of assets in employer stock.
"
"All of the following can be used to determine a participant's benefit in a defined benefit plan, EXCEPT:
A. Career average compensation
B. Plan’s benefit formula
C. Interest and dividends
D. Final average compensation
E. Years of service
" "The correct answer is C.
This is false because interest and dividends are not used to determine a participant’s benefit in a defined benefit plan.
"
"All of the following are normal forms of benefit payment from a defined benefit plan, EXCEPT:
A. Life only annuity
B. Lump sum
C. Joint and survivor annuity
D. Life annuity with ten years certain
E. Straight life annuity
" "The correct answer is B.
This is false because while the plan may allow a lump-sum distribution, it is not the normal form of payment found in a defined benefit plan.
"
"Which of the following statements regarding cash balance plans is/are TRUE?
I. The sum of the contributions and interest credits at any period of time are referred to as a theoretical account balance.
II. An actuary must certify the contribution requirements.
III. The employer bears the risk of investment.
A. I only
B. II only
C. I and III only
D. II and III only
E. I, II and III
" "The correct answer is E.
All statements are true.
"
"All of the following statements regarding defined benefit plans are TRUE, EXCEPT:
A. After-tax employee contributions may be accepted however they are not considered part of the funding for the defined benefit.
B. Mandatory employee contributions may occur in state-sponsored governmental plans and a portion of the defined benefit is funded by the mandatory employee contributions.
C. Employer contributions typically fund all of the employees’ benefits.
D. Investments that earn less than the assumed interest rate will cause the employer contribution to be less.
E. Older participants will tend to have higher contribution requirements than younger participants.
" "The correct answer is D.
This statement is false because if the investments earn less than the interest assumption it will cause the employer to make a higher contribution to fund th benefits rather than a lower one.
"
"All of the following are retirement benefit formulas for a defined benefit plan, EXCEPT:
A. Offset formula
B. Excess formula
C. Flat percentage of compensation formula
D. Unit credit formula
E. Accrued benefit formula
" "The correct answer is E.
This is false because an accrued benefit formula does not determine the retirement benefit. It is the portion of the normal retirement benefit that the participant has earned at a particular date.
"
"All of the following statements regarding cash balance plans are TRUE, EXCEPT:
A. The plan is required to offer an annuity form of benefit.
B. The PBGC guarantees at least a portion of the participant’s benefits in the event of employer bankruptcy or other insolvency if PBGC covered.
C. If the investment rate of return is lower than the guaranteed interest credit then the employer contribution is increased.
D. The plan defines the guaranteed benefit in the form of a monthly benefit payable at normal retirement age.
E. The participant may elect to receive a lump sum distribution if offered by the plan.
" "The correct answer is D.
This statement is false because the participants benefit is defined in terms of an account balance rather than a benefit formula.
"
"Which of the following statements regarding the normal form of benefit payment
in a defined benefit plan is/are TRUE?
I. A life annuity with ten years certain pays a monthly benefit to the participant for life then guarantees payment to the spouse for ten additional years.
II. A life annuity provides for monthly payment of benefits for the lifetime of the participant.
III. A joint and survivor annuity pays a monthly benefit over the life of the participant and ceases upon the participant death.
A. I only
B. II only
C. I and III only
D. II and III only
E. I, II and III only
" "The correct answer is B.
Statement I is false because a life annuity with ten years certain pays a monthly benefit to the participant for life and guarantees ten years of payments. If the participant dies prior to the end of the ten-year period then the beneficiary will receive the remaining payments. Statement III is false because a joint and survivor annuity pays a monthly benefit over the life of the participant and then pays a benefit to the beneficiary until the beneficiary’s death. The survivor annuity is typically 50%, 75% or 100% of the monthly payment to the participant.
"
"All of the following are retirement benefit formulas that may be used in a defined benefit plan, EXCEPT:
A. $500 per month
B. $25 per month times years of service
C. 75% of final average compensation
D. 5% of monthly compensation times years of service
E. 10% of annual profits
" "The correct answer is E.
This is false because E is not a defined benefit formula. The benefit formulas are not based on the employer’s profits.
"
"Which of the following is/are compensation definitions used to determine a participant’s benefit under a defined benefit plan?
I. Compensation earned during the month prior to retirement.
II. Compensation earned during the participant’s working lifetime with the employer.
III. An average of compensation earned by the participant over a specified period.
A. I only
B. III only
C. I and II only
D. II and III only
E. I, II and III
" "The correct answer is D.
The normal bond amount is the greater of $1,000 or 10% of plan assets up to a maximum of $500,000. Since the plan invests in employer securities, the maximum bond amount of $500,000 is increased to $1,000,000. 10% of plan assets is $3,155,000 ($31,550,000 *.10) which is more than $1,000,000. Therefore, the bon amount is $1,000,000.
"
"Which of the following statements regarding cash balance plans is/are TRUE?
I. It is a defined benefit plan that defines the guaranteed benefit in the form of an account balance rather than a monthly benefit payable at normal retirement age.
II. The employee bears the risk of investment.
III. The annuity form of benefit may not exceed the IRC §415 maximum benefit limitation for defined benefit plans.
A. I only
B. II only
C. I and III only
D. II and III only
E. I, II and III
" "The correct answer is A.
Statements II and III are false. A master plan, prototype plan or volume submitter plan may consist of the two-part document: basic plan document an adoption agreement. The individually designed plan provides the greatest amount of design flexibility.
"
"Based on the following information, determine the amount of the fidelity bond:
Profit sharing plan with 200 participants
Total assets equal $10,850,000
There are no assets invested in employer stock
A. $1,000
B. $18,500
C. $185,000
D. $500,000
E. $1,085,000
" "The correct answer is D.
The normal bond amount is 10% of plan assets up to a maximum of $500,000. 10% of plan assets is more than $500,000, and there were no employer stock investments therefore the plan only need provide a bond of $500,000.

"
"Based on the following information, determine the amount of the fidelity bond necessary to satisfy the small plan audit waiver requirements:
Profit sharing plan with 75 participants
Total assets equal $4,000,000
Amount in qualifying assets equal $3,000,000
A. $ 400,000
B. $ 500,000
C. $1,000,000
D. $4,000,000
E. $7,000,000

" "The correct answer is C.
The normal bond amount is 10% of plan assets up to a maximum of $500,000. However, in order to satisfy the small plan audit requirements, nonqualifying assets must be completely bonded. The amount of nonqualifying assets is $1,000,000 ($4,000,000 - $3,000,000) so the bond needs to be for that amount.
"
"All of the following are employee documents that may be needed when an employer is adopting a new participant-directed 401(k) plan, EXCEPT:
A. Beneficiary designation
B. Rollover request form
C. Salary reduction agreement
D. Investment elections
E. Birth certificate
" "The correct answer is E.
This is false because the participant birth certificate is not needed when setting up a new plan.
"
"Based on the following information, determine the maximum deductible contribution to the profit sharing plan:
Total compensation for all eligible participants is $500,000.
The profit sharing plan is the only retirement plan sponsored by the employer.
No one exceeds the IRC §401(a)(17) compensation limit.
A. $0
B. $50,000
C. $75,000
D. $125,000
E. $500,000
" "The correct answer is D.
The maximum deductible contribution to a profit sharing plan is 25% of eligible compensation. The total compensation for eligible participants is $500,000 so the maximum deductible contribution is $125,000 ($500,000 x 25%).
"
"All of the following are characteristics of prototype plans, EXCEPT:
A. No pooling of plan assets among employers
B. More flexible than individually designed plans
C. Includes an adoption agreement
D. Pre-approved plan document language
E. Includes a basic plan document
" "The correct answer is B.
This is false because individually designed plans are more flexible than prototype plans.
"
"All of the following statements regarding fidelity bond requirements are TRUE, EXCEPT:
A. Generally, the maximum bond amount is $500,000.
B. Generally, the minimum bond amount is $1,000.
C. A bond is not required if the owner is the only participant.
D. The bond amount is based on plan assets.
E. A large plan filer may need a bond in excess of the maximum to satisfy additional audit requirements.
" "The correct answer is E.
This statement is false because a large plan filer does not need an additional bond amount to satisfy additional audit requirements. This applies to small plan filers.
"
"All of the following are supporting documents that are needed when implementing a new plan for a corporation with 50 employees, EXCEPT:
A. Preparation of a resolution to adopt the plan
B. Preparation of a plan amendment
C. Preparation of a QDRO policy
D. Preparation of an SPD
E. Application for a fidelity bond
" "The correct answer is B.
This is false because preparation of a plan amendment is not needed when implementing a new plan.
"
"Based on the following information, determine the maximum deductible contribution to the 401(k) plan for 2014:
Total gross compensation for the 100% owner is $300,000.
Total gross compensation for all eligible employees is $650,000.
The 401(k) plan is the only plan sponsored by the employer.
The employees have made pre-tax elective contributions of $75,000.
The compensation limit for 2014 is $260,000.
A. $162,500
B. $227,500
C. $237,500
D. $260,000
E. $312,500
" "The correct answer is B.
The maximum deductible contribution to a 401(k) plan is 25% of eligible compensation. Compensation for a person shall not exceed the IRS §401(a)(17) limit of $260,000 for 2014. Since the owner has compensation in excess of this amount, his compensation shall be limited to $260,000. The total compensation for all eligible participants is $910,000 ($260,000 + $650,000). The maximum deductible contribution is $227,500 ($910,000 x 25%). The pre-tax elective contributions are not included in the maximum deductible contributions since they are separately deductible.
"
"Based on the following information, determine the amount of the fidelity bond required for a 401(k) plan:
There are 750 participants
Total assets equal $31,550,000
Assets invested in employer stock equal $859,000
A. $1,000
B. $500,000
C. $859,000
D. $1,000,000
E. $3,155,000
" "The correct answer is D.
The normal bond amount is the greater of $1,000 or 10% of plan assets up to a maximum of $500,000. Since the plan invests in employer securities, the maximum bond amount of $500,000 is increased to $1,000,000. 10% of plan assets is $3,155,000 ($31,550,000 *.10) which is more than $1,000,000. Therefore, the bond amount is $1,000,000.
"
"Which of the following statements regarding plan documents is/are TRUE?
I. All plans using a master plan must invest their funds in a single master trust.
II. An individually designed plan consists of the basic plan document and the adoption agreement.
III. The volume submitter plan provides the greatest amount of design flexibility.
A. I only
B. II only
C. I and III only
D. II and III only
E. I, II and III
" "The correct answer is A.
Statements II and III are false. A master plan, prototype plan or volume submitter plan may consist of the two-part document: basic plan document and adoption agreement. The individually designed plan provides the greatest amount of design flexibility.
"
"All of the following are important provisions of the plan document,, EXCEPT:
A. Vesting
B. Allocations or benefits
C. Top-heavy
D. Normal Retirement
E. Prohibited transactions
" "The correct answer is E.
This is false because there is no requirement to have a listing of prohibited transactions in the document.
"
"All of the following statements regarding the definition of retirement found in a plan document are TRUE, EXCEPT:
A. A plan may define NRA as the attainment of age 73.
B. Eligibility provisions for early retirement may include specific age and/or service requirements.
C. In general, under a pension benefit plan a participant must separate from service to obtain early retirement benefits.
D. Late retirement occurs when a participant elects to retire later than the plan’s NRA.
E. NRD may be defined as the exact date a participant attains NRA.
" "The correct answer is A.
This statement is false because the maximum NRA allowable by the IRC is the greater of attainment of age 65, or if later, the fifth anniversary of the participant’s initial date of plan participation. Therefore, a plan may not define an age past 65.
"
"All of the following are vesting schedules that may be used in a defined benefit plan, EXCEPT:
A. 100% immediate
B. Three-year cliff
C. Seven-year graded
D. Five-year cliff
E. Ten-year cliff
" "The correct answer is E.
This is false because a defined benefit plan must provide a vesting schedule that is as rapid as the 7 year graded or the 5-year cliff in all years of the schedule. A ten-year cliff schedule does not meet the minimum vesting percentages for either schedule.
"
"Based on the following information, determine the vested portion of the
participant’s account:
The plan is a money purchase pension plan that does not allow for rollover or after-tax employee contributions.
The participant’s account balance is $40,000.
The participant is 40% vested.
A. $1,600
B. $10,000
C. $16,000
D. $24,000
E. $40,000
" "The correct answer is C.
The $40,000 account balance is subject to 40% vesting. The participant’s vested portion (the portion owned) is $16,000 ($40,000 x 40%).
"
"Based on the following information, determine the employee’s entry date:
• The plan is a calendar year plan.
• The plan’s eligibility requirements are age 21 and one year of service.
• One year of service is defined as a 12-month period in which an employee works 1,000 hours of service.
• The entry dates are the first day of the plan year or the first day of the seventh month of the plan year following when the eligibility requirements are met.
• The employee has worked 1,000 hours during his first year of employment.
• The employee’s date of birth is September 21, 1993.
• The employee’s date of hire is February 4, 2014.
A. July 1, 2014
B. September 21, 2014
C. January 1, 2015
D. February 3, 2015
E. July 1, 2015
" "The correct answer is E.
The employee attains age 21 on September 21, 2014 and completes a year of service on February 3, 2015. The later of these two dates is February 3, 2015. The plan allows for two entry dates; January 1st and July 1st. The employee will enter the plan on July 1, 2015 (the entry date coincident with or next following February 3, 2015).
"
"All of the following statements regarding vesting are TRUE, EXCEPT:
A. Pre-tax elective contributions are always 100% vested immediately.
B. Vesting must be 100% upon death.
C. A defined contribution plan is required to use a vesting schedule at least as favorable as the six-year graded or the three-year cliff vesting schedules for employer contributions.
D. Vesting must be 100% upon attainment of normal retirement age.
E. Matching contributions must vest at least as favorably as the six-year graded or the three-year cliff vesting schedules.
" "The correct answer is B.
This statement is false because while common, vesting need not be 100% upon death.
"
"Based on the following information, determine the participant’s vested percentage as of December 31, 2014:
• A vesting year of service is defined as a calendar year in which the participant worked more than 1,000 hours of service.
• The vesting schedule is a six-year graded schedule.
• The plan is a profit sharing plan effective March 1, 1991.
• The plan’s NRD is age 65.
• The participant is a fulltime employee who was hired January 20, 2011.
• The participant’s date of birth is October 25, 1976.
A. 20%
B. 40%
C. 60%
D. 80%
E. 100%
" "The correct answer is C.
The participant has worked more than 1,000 hours in 2011, 2012, 2013 and 2014. As of December 31, 2014 the participant has been credited with four years of vesting service. Using a six-year graded vesting schedule, the participant is 60% vested. However, it is also necessary to check to see if the participant has reached NRA. If so, the vested percentage would be accelerated to 100%. Based on the facts in the question the participant has not reached NRA, therefore the vested percentage is 60%.

"
"Based on the following information, determine the participant’s eligibility date:
• The plan is a calendar year plan.
• The plan’s eligibility requirements are age 21 and one year of service.
• One year of service is defined as a 12-month period in which an employee


works 1,000 hours of service.
• The employee has worked 1,000 hours during his first year of employment.
• The employee’s date of birth is April 15, 1994.
• The employee’s date of hire is November 1, 2014.
A. November 1, 2014
B. January 1, 2015
C. April 15, 2015
D. November 1, 2015
E. January 1, 2016

" "The correct answer is D.
The data states that an employee must meet a one year of service requirement and attain age 21 to be eligible. The employee will be age 21 on April 15, 2015 and complete one year of service on October 31, 2015. Thus on November 1, 2015 the employee will have met both eligibility requirements.
"
"Based on the following information, determine when Participant A will reach NRA:
• The plan uses the maximum NRA allowed under the IRC and ERISA.
• Participant A’s date of birth is July 1, 1951.
• Participant A's date of hire was June 1, 2007.
• Participant A's date of plan participation was July 1, 2007.
• The plan year end is June 30th.
A. January 1, 2011
B. July 1, 2011
C. January 1, 2012
D. July 1, 2012
E. July 1, 2016
" "The correct answer is E.
The maximum NRA under the plan cannot be greater than 1) the time a participant attains age 65, or if later 2) the fifth anniversary of the participant’s initial date of plan participation. Participant A attains age 65 on July 1, 2016. Participant A has five years of participation on July 1, 2012. Therefore, the maximum NRA for Participant A is July 1, 2016.
"
"Which of the following is/are years of service that may be excluded in the computation of vesting?
I. Years of service earned prior to age 18.
II. Years of service earned prior to the inception date of the plan.
III. Years of service prior to date of participation.
A. I only
B. III only
C. I and II only
D. II and III only
E. I, II and III
" "The correct answer is C.
Statement III is false because service prior to the date of participation is not a service period definition that can be excluded.
"
"All of the following statements regarding ERISA are TRUE, EXCEPT:
A. It provided rules to determine when a participant is vested in benefits.
B. It established plan termination insurance.
C. It established IRAs.
D. It established educational requirements for actuaries.
E. It mandated retirement benefits for all workers.
" "The correct answer is E.
This statement is false because ERISA did not mandate retirement benefits for all workers.
"
"All the following guidance can be issued by the IRS, EXCEPT:
A. Legislative regulations
B. Corrective regulations
C. Temporary regulations
D. Final regulations
E. Proposed regulations
" "The correct answer is B.
This is false because the IRS does not issue corrective regulations.
"
"All of the following statements are general requirements that must be met for a plan to be qualified, EXCEPT:
A. It must be communicated to employees.
B. It must be established to include all former employees.
C. It must be intended to be permanent.
D. It must be in writing.
E. It must be for the exclusive benefit of the employees.
" "The correct answer is B.
This statement is false because a qualified plan need not be established for all employees or for former employees.
"
"All of the following guidance may be issued by the DOL, EXCEPT:
A. Determination letters
B. Advisory opinions
C. News releases
D. Labor regulations
E. Information letters
" "The correct answer is A.
This statement is false because determination letters are issued by the IRS, not the DOL.
"
"Which of the following statements regarding the enforcement of laws affecting retirement plans is/are TRUE?
I. IRS enforces Title 1 of ERISA rules that relate to participation, funding and vesting.
II. DOL enforces Title 1 of ERISA rules that relate to reporting, disclosure and standards of conduct.
III. PBGC enforces Title 1 of ERISA rules that relate to crediting or accrual of benefits and the method and payment of benefits.
A. I only
B. III only
C. I and II only
D. II and III only
E. I, II and III
" "The correct answer is C.
Statement III is false. The PBGC was established under Title IV of ERISA. Its responsibility relates to the functions of Title IV, not Title I. Title IV provides for a termination insurance program that applies only to defined benefit plans.
"
"All of the following statements regarding the order of authority for guidance issued by the IRS are TRUE, EXCEPT:
A. Revenue Rulings have more authority than revenue procedures.
B. Federal court decisions have more authority than state law.
C. Laws enacted by Congress have more authority than IRS PLRs.
D. Field Assistance Bulletins have more authority than Revenue Rulings.
E. General Counsel Memorandums have more authority than IRS PLRs.
" "The correct answer is D.
This statement is false because Revenue Rulings have more authority than Field Assistance Bulletins.
"
"All of the following must be specified in the plan document for a plan to be qualified, EXCEPT:
A. The limit on the amount of compensation that can be taken into account for determining contributions
B. A limit on the level of contributions to a participant
C. Plan must not discriminate against lower-paid employees
D. Benefits earned by a participant may not be assigned.
E. Contributions allocated are 100% immediately vested upon termination of employment.
" "The correct answer is E.
This is false because a plan does not have to force 100% immediate vesting upon termination of employment. The plan would force 100% vesting if the participants were at normal retirement age or if the plan were to terminate.
"
"Which of the following is/are benefits of sponsoring a qualified plan?
I. Interest income on funds within the plan’s trust are tax-exempt.
II. Distributions from the plan are made tax-free.
III. Benefits under the plan typically accumulate on a tax-deferred basis.
A. II only
B. III only
C. I and II only
D. I and III only
E. I, II and III
" "The correct answer is D.
Statement II is false because distributions to participants are generally taxable. However, the portion of the distribution that consists of after-tax employee contributions or designated Roth contributions is not taxed. In addition, if the participant rolls the distribution into another qualified plan or IRA the taxes will be deferred to a later date.
"
"All of the following must be specified in the plan document for a plan to be qualified, EXCEPT:
A. Benefits may be assigned to participant creditors
B. The maximum period of time before a participant is granted ownership of benefits
C. In some plans, payment of benefits in the form of a qualified joint and survivor annuity
D. Cover a required minimum number of employees
E. Top-heavy plans must satisfy faster vesting
" "The correct answer is A.
This is false because a plan must specify that benefits earned by a participant may not be assigned or alienated. They must not be granted to another party or attached by outside parties, such as the participant or the plan sponsor’s creditors.
"
"Which of the following is/are receiving favorable tax treatment from a qualified plan?
I. The sponsor
II. The participants
III. The trust
A. II only
B. III only
C. I and II only
D. I and III only
E. I, II and III
" "The correct answer is E.
All statements are true.
"
"All of the following must be addressed in the SPD, EXCEPT:
A. Eligibility requirements
B. Contribution allocation provisions
C. Distribution of benefits
D. Submission date of the SPD to the IRS
E. Vesting schedules
" "The correct answer is D.
This is false because SPDs are not submitted to the IRS.
"
"All of the following may be provided to new employees upon their becoming eligible to participate in a participant-directed investment 401(k) plan using a QDIA, EXCEPT:
A. Beneficiary designation election
B. Tax withholding information
C. Investment direction election
D. Salary reduction agreement
E. QDIA notice
" "The correct answer is B.
This is false because tax withholding forms are provided to employees upon distribution of benefits not at eligibility.
"
"Which of the following contribution amounts is/are designated to be contributed to the plan on the salary reduction election?
I. Nonelective contribution.
II. Pre-tax elective contribution.
III. Designated Roth contribution.
A. I only
B. II only
C. I and III only
D. II and III only
E. I, II and III
" "The correct answer is D.
Item I is false since nonelective contributions are not employee contributions. They are profit sharing contributions made by the employer.
"
"Which of the following statements regarding the notice given to a participant who has been automatically enrolled in a 401(k) plan is /are TRUE?
I. It must explain the participant’s right to choose a different pre-tax elective contribution rate.
II. It must explain how the pre-tax elective contribution will be invested if the participant does not make an investment election.
III. It must explain the participant’s right to elect not to make a pre-tax elective contribution.
A. I only
B. II only
C. I and III only
D. II and III only
E. I, II and III
" "The correct answer is E.
All statements are true.
"
"All of the following plans may roll over a distribution into another qualified plan, EXCEPT:
A. 403(b) plan
B. Profit sharing plan
C. SEP
D. Nonqualified plan
E. 457(b) plan
" "The correct answer is D.
This is false because a qualified plan may not accept a rollover from a nonqualified plan.
"
"All of the following statements regarding rollovers are TRUE, EXCEPT:
A. A separate account is set up and maintained for the employee’s rollover account.
B. A plan is not required to accept rollovers from other plans.
C. The participant pays taxes on the amounts rolled over.
D. The rollover account is fully vested at all times.
E. A plan sponsor may determine if the rollover came from an acceptable source by looking up the plan on the DOL’s EFAST2 website
" "The correct answer is C.
This statement is false because the participant continues to defer taxes on the amounts rolled over.
"
"Which of the following is/are provided to enroll a new participant in a profit sharing plan?
I. A summary plan description
II. A beneficiary designation
III. An IRA prospectus
A. II only
B. III only
C. I and II only
D. I and III only
E. I, II and III
" "The correct answer is C. §§8.04 [A] and 8.05 [C] and [D]
Item III is false because an IRA prospectus is not given to a participant in a profit sharing plan.
"
"Which of the following statements regarding the notice given under an automatic contribution arrangement is/are TRUE?
I. It must be given to new participants a reasonable period of time before the pre-tax elective contribution initially begins.
II. It must be given to all employees annually before each plan year.
III. It must be given to explain the employer’s automatic profit sharing contribution that will be made by the employer at year end.
A. I only
B. II only
C. I and II only
D. II and III only
E. I, II and III
" "The correct answer is A.
Statement II is false because the notice is given to all affected employees not all employees. Statement III is false because the notice communicates information about the automatic pre-tax elective contribution rate and not about the employer’s profit sharing contribution.
"
"Which of the following statements regarding rolling over assets into another qualified plan is/are TRUE?
I. A designated Roth 401(k) that includes amounts that are not taxable can be rolled to another 401(k) plan that allows designated Roth accounts if a direct rollover occurs.
II. After a rollover occurs it is subject to the rules and regulations of that plan.
III. The plan and SPD will specify whether or not rollover contributions are allowed.
A. II only
B. III only
C. I and II only
D. I and III only
E. I, II and III
" "The correct answer is E.
All statements are true.
"
"Which of the following is/are provided to enroll a new participant in a 401(k) plan?
I. Salary reduction election
II. Summary plan description
III. Corporate resolution adopting the plan
A. I only
B. III only
C. I and II only
D. II and III only
E. I, II and III
" "The correct answer is C.
Statement III is false because a corporate resolution to adopt the plan is not provided to the participants to enroll them in the plan.
"
"All of the following are methods that may be used to provide information to participants, EXCEPT:
A. Hand delivery
B. First class mail
C. Inclusion in a regularly provided newsletter
D. Posting on a workplace bulletin board
E. Electronic delivery for certain disclosures
" "The correct answer is D.
This is false because posting on a workplace bulletin board is not an acceptable method of delivering information to participants.
"
"All of the following statements regarding the electronic disclosure of information to participants are TRUE, EXCEPT:
A. The plan administrator must confirm that documents delivered electronically have been received.
B. All documents transmitted electronically must be protected with password sensitive encryption.
C. Participant confidentiality must be protected.
D. A notice of the significance of the document being transmitted electronically must be provided.
E. The participant must be advised of the right to receive the information by paper.
" "The correct answer is B.
This statement is false because all documents transmitted electronically need not be protected with password sensitive encryption. However, some may to protect participant confidentiality.
"
"All of the following statements regarding Form 5500 are TRUE, EXCEPT:
A. Generally, it is due seven months after the end of the plan year.
B. The Form 5500 to be filed relates to the year in which the plan year ends.
C. Generally, a small plan filer is one that has fewer than 100 participants at the beginning of the plan year.
D. The IRS and the DOL share the information on Form 5500.
E. A 2½ month extension of time to file Form 5500 may be requested by filing Form 5558.
" "The correct answer is B.
This statement is false because the Form 5500 to be filed relates to the year in which the plan year begins.
"
"All of the following are disclosures that must be provided to participants of a safe harbor 401(k) plan with automatic enrollment, EXCEPT:
A. SAR
B. Participant statements
C. Bylaws
D. Safe harbor notice
E. Automatic enrollment notice
" "The correct answer is C
This is false because bylaws are not required to be given to participants.
"
"All of the following statements regarding plan audits by an independent public accountant are TRUE, EXCEPT:
A. Plans that cover 100 or more participants are typically required to include a plan audit report with their Form 5500 series filing.
B. A plan audit must provide an opinion as to whether the plan design meets the overall objective of the plan sponsors.
C. Plans that have fewer than 100 participants and file a Schedule H are required to file a plan audit report with their Form 5500 series filing.
D. A plan audit may address whether plan assets have been fairly valued.
E. Plans that have fewer than 100 participants and file a Schedule I may elect to waive the plan audit report if they meet certain requirements.
" "The correct answer is B.
This statement is false because a plan audit does not provide an opinion as to whether the plan design meets the overall objective of the plan sponsors.
"
"Which of the following statements regarding Form 8955-SSA is/are TRUE?
I. It is used to report deferred vested benefits to the IRS.
II. It must be filed by the last day of the seventh month following the plan year end (plus extensions).
III. It contains a listing of the participants who have terminated employment and are still due a vested benefit from the plan.
A. I only
B. III only
C. I and II only
D. II and III only
E. I, II and III
" "The correct answer is E.
All statements are true.
"
"Based on the following information, all of the following may be filed with Form
5500, EXCEPT:
• The plan is a 401(k) plan.
• There are 455 participants.
• $10,000 in fees was paid by the plan for administration.
A. Schedule C
B. Schedule G
C. Schedule H
D. Schedule SB
E. Accountant’s Report
" "The correct answer is D.
This is false because Schedule SB is only filed if the plan is a defined benefit plan.
"
"All of the following are events that require disclosures to plan participants, EXCEPT:
A. Receipt of sales inventory
B. Distribution request
C. Enrollment
D. Loan request
E. Receipt of a domestic relations order
" "The correct answer is A.
This is false because receipt of sales inventory does not generate retirement plan
disclosures.
"
"All of the following may be found on participant statements, EXCEPT:
A. Vested account balance
B. Contributions allocated
C. An explanation of any permitted disparity
D. An explanation of any floor-offset arrangement
E. An explanation of a QDRO
" "The correct answer is E.
This is false because information about QDROs is not found on the participant statement. Upon receipt of a DRO the plan is required to inform the affected participants.
"
"Based on the following information, all of the following may be filed to comply with the Annual Return/Report filing, EXCEPT:
• The plan is a single employer defined benefit plan.
• There are 45 participants.
• $10,000 in fees was paid by the plan for administration.
• The plan contains life insurance policies.
A. Schedule A
B. Schedule G
C. Schedule I
D. Schedule SB
E. Form 5500-SF
" "The correct answer is B.
This is false because Schedule G is not required to be filed by a small plan.
"
"All of the following statements regarding withdrawals are TRUE, EXCEPT:
A. A 401(k) plan may allow withdrawals after a participant has reached age 59½.
B. A profit sharing plan may offer withdrawals on funds that have been held in the plan at least two years.
C. A defined benefit plan may offer withdrawals to participants who have attained age 62.
D. A money purchase plan may offer withdrawals if a participant has been a participant for 6 years.
E. An ESOP may offer withdrawals in order to satisfy the diversification requirements.
" "The correct answer is D.
This statement is false because a money purchase plan may not offer withdrawals just for length of time. The participant must attain age 62 to receive a withdrawal.
"
"All of the following statements regarding hardship withdrawals are TRUE, EXCEPT:
A. The plan’s hardship provisions must be defined.
B. The definition of hardship must be applied in a uniform manner.
C. The amount available must be determined in a uniform manner.
D. Hardship withdrawals must be offered in all 401(k) plans.
E. The amount of hardship withdrawal cannot exceed the lesser of the amount available from the participant’s account or the immediate need.
" "The correct answer is D.
This statement is false because hardship withdrawals are a plan design option and not a requirement in 401(k) or any other retirement plan.
"
"All of the following describe allowable withdrawals, EXCEPT:
A. Withdrawal of after-tax employee contributions
B. Withdrawal of rollover contributions
C. Withdrawal at NRA
D. Withdrawal of employer profit sharing contributions deposited two years ago
E. Withdrawal of employer contributions from a money purchase plan by a current employee prior to age 62
" "The correct answer is E.
This is false because employer contributions from a money purchase plan are not available for withdrawals prior to age 62.
"
"All of the following statements regarding the 401(k) hardship general standards are TRUE, EXCEPT:
A. The general standard has two parts, the events test and the needs test.
B. A participant must have an immediate and heavy financial need to be granted the withdrawal.
C. Purchase of an automobile to commute to work may be considered a hardship event.
D. The amount of the need may include estimated taxes and penalties.
E. Resources of the participant’s adult children must also be considered in determining whether to make the hardship withdrawal.
" "The correct answer is E.
This statement is false because resources of the participant’s adult children are not considered in determining whether to allow the hardship withdrawal. However resources of minor children would be considered.
"
"All of the following incidents are 401(k) hardship safe harbor withdrawal events, EXCEPT:
A. To prevent eviction of the participant from the participant’s principal residence
B. Tuition for the participant’s child’s doctorate program
C. Medical expenses for the participant’s spouse
D. Expenditures necessary to acquire a second home
E. Room and board for college expenses of the participant
" "The correct answer is D.
This is false because only payment of expenditures necessary to acquire a primary home is a safe harbor hardship withdrawal event. A secondary home is not included.
"
"Which of the following statements regarding restrictions that may be imposed on hardship withdrawals for plans that are not 401(k) types is/are TRUE?
I. The type of money available for a hardship withdrawal may be limited.
II. Pension plans only allow hardship withdrawals if the participant has reached NRA or attained age 62.
III. The plan must define the financial circumstances that constitute a hardship event.
A. I only
B. II only
C. I and III only
D. II and III only
E. I, II and III
" "The correct answer is E.
All statements are true.
"
"Which of the following is/are 401(k) hardship safe harbor standard needs requirements?
I. The participant is not permitted to make elective contributions for at least six months following the date the hardship withdrawal is received.
II. The participant must withdraw 10% more than the need to cover any incidental costs that may be incurred.
III. The participant has obtained all nontaxable loans from all plans maintained by the employer prior to the hardship withdrawal (unless they increase the hardship).
A. I only
B. II only
C. I and III only
D. II and III only
E. I, II and III
" "The correct answer is C.
Statement II is false because the participant may only withdraw the lesser of the amount necessary to relieve the financial need (plus estimated taxes and penalties) or the amount available from the participant’s account. No additional amount may be withdrawn and it is not required that the hardship amount include estimated taxes and penalties.
"
"All of the following steps must be taken by the plan administrator when a DRO is received, EXCEPT:
A. Notify the participant that no further allocations will be made to the participant account until the QDRO is paid out.
B. Notify the participant of the receipt of the DRO.
C. Notify the participant and alternate payee in writing whether the DRO is a QDRO.
D. Notify the participant and alternate payee of the procedures for determining the DRO’s qualified status.
E. Notify the alternate payee of the receipt of the DRO.
" "The correct answer is A.
This is false because the plan is required to separately account for the amounts that would have been payable to the alternate payee during the period that the QDRO is being reviewed to see if it meets the requirements of a QDRO. There is no requirement that allocations and earning to the account balance stop being allocated.
"
"All of the following incidents are 401(k) hardship safe harbor withdrawal events, EXCEPT:
A. Funeral expense for the children
B. Repair of the engine for the participant’s boat
C. Roof repair paid for damage to the participant’s principal residence due to a tornado
D. Closing costs paid for the participant’s principal residence
E. Electronic books purchased for the children’s college education
" "The correct answer is B.
This is false because the repair of the boat engine is not considered a safe harbor hardship withdrawal event.
"
"All of the following may be alternate payees, EXCEPT:
A. Brother who is not a dependent
B. Daughter
C. Son
D. Former Spouse
E. Dependent parent
" "The correct answer is A
This is false because an alternate pay may include a dependent brother but not an independent brother.
"
"All of the following must clearly be described in a loan program, in a manner understandable to the average plan participant, EXCEPT:
A. The method by which a participant may obtain a determination letter for the plan
B. The basis on which loans are approved or denied
C. Limitations, if any, on the number of loans allowed
D. The type of collateral that may be used to secure the loan
E. The events that constitute a loan default
" " The correct answer is A.
This is false because the determination letter for the plan need not be described in the loan program.
"
"Based on the following information, determine the maximum loan amount that Participant C can take on December 31, 2014:
• The plan allows two outstanding loans.
• Participant C’s vested balance as of December 31, 2014 is $60,000.
• Participant C has an outstanding loan, which had a balance of $10,000 on December 31, 2013.
• The balance of Participant C’s outstanding loan is $6,000 on December 31, 2014.
A. $6,000
B. $10,000
C. $24,000
D. $30,000
E. $50,000
" The correct answer is C.

"All of the following statements are requirements that must be met in a participant loan program in order to avoid a PT, EXCEPT:
A. Loans must be made available to participants on a reasonably equivalent basis.
B. Loans must not be made available to HCEs in an amount greater than the amount available to NHCEs.
C. The plan document must contain the terms of the loan program, or the employer must adopt a formal loan policy or procedure.
D. All loans must bear a reasonable rate of interest.
E. Loans may not be secured by either the participant’s vested benefit or other collateral.
" "The correct answer is E.
This statement is false because loans must be secured by either the participant’ vested benefit or other collateral.
"
"Based on the following information, determine the maximum loan amount tha Participant K can take on December 31, 2014:
• The plan allows two loans at one time.
• The Participant does not have a loan.
• The Participant would like to take the maximum loan amount available.
• The Participant’s vested account balance as of December 31, 2014 is $40,000.
A. $0
B. $10,000
C. $20,000
D. $40,000
E. $50,000
" The correct answer is C.

"Based on the following information, determine the maximum loan amount that Participant M can take on December 31, 2014:
• The plan allows one outstanding loan at a time.
• Participant M’s account balance as of December 31, 2014 is $12,000.
• Participant M’s vested account balance as of December 31, 2014 is $8,000.
• Participant M does not have an outstanding loan.
• The plan allows only plan assets as collateral for the loan.
A. $4,000
B. $6,000
C. $8,000
D. $10,000
E. $50,000
" The correct answer is A.

"All of the following statements are requirements that must be met in a participant loan program in order to avoid current taxation, EXCEPT:
A. The loan is actually repaid according to the loan terms.
B. The term of the loan is not too long.
C. The amount of the loan is not too much.
D. The loan is limited to the value of the account balance.
E. The repayment schedule for the loan meets the IRC requirements.
" "The correct answer is D.
This statement is false because the loan is limited the lesser of a or b:
a. $50,000, reduced by any excess of 1 minus 2:
1. The highest outstanding balance of loans during the 12 months ending on the day before the date on which the loan is made; minus
2. The outstanding balance of loans on the date on which the loan is made.
b. The greater of:
1. 50% of the participant’s vested account balance or benefit; or
2. $10,000, if allowed by the plan.
"
"Which of the following is/are included in a loan policy?
I. Application procedures
II. Procedure for determine a reasonable rate of interest
III. Identity of the person or position administering the loan program
A. I only
B. III only
C. I and II only
D. II and III only
E. I, II and III
" "The correct answer is E.
Correct because all statements are true.
"
"Which of the following is/are included in a loan policy?
I. Procedure for paying off a loan early
II. Steps that must be taken to preserve plan assets in the event of loan default
III. Name of each participant that has an outstanding loan
A. I only
B. II only
C. I and III only
D. II and III only
E. I, II and III
" "The correct answer is B.
Statement I is false because the procedures for paying off a loan early are not part of the required items to be included in a loan policy. Statement III is false because the names of the participants that have an outstanding loan is not an item that must be included in the loan policy.
"
"Which of the following statements regarding determining the interest rate for a loan is/are TRUE?
I. The IRC and ERISA do not specifically contain a safe harbor method for determining a reasonable rate of interest.
II. The interest rate may be defined in the loan policy to be a specific fixed rate such as 8%, which will be the same percent for all loans.
III. The interest rate on a loan issued to a participant on military leave may be reduced to 6% during the period of military service.
A. II only
B. III only
C. I and II only
D. I and III only
E. I, II and III
" "The correct answer is D.
Statement II is false because the rate cannot be a set fixed percentage for all loans. It must reflect a current rate charged for similar loans by financial lending institutions in the community at the time the loan is taken out. These rates fluctuate often. Therefore a fixed rate that is the same for all loans would not be reasonable.
"
"Based on the following, what is/are the maximum loan amount the participant may borrow?
• The participant’s account balance is $13,000.
• The participant’s vested account balance is $5,200.
• The participant does not have any other loans.
I. $10,00 if the plan allows outside collateral
II. $2,600 if the plan does not allow outside collateral
III. $50,000 if the plan allows outside collateral
A. I only
B. II only
C. I and II only
D. II and III only
E. I, II and III
" "The correct answer is C.
Statement III is false because the loan under the facts described is limited to the lesser of a) $50,000 or b) 50% of the participant’s vested account balance ($2,600) or the $10,000 de minimis amount. The lesser of a) or b) is $10,000 if the plan allows for outside collateral. If it does not, then the limit is $2,600.
"
"All of the following describe distributable events that may be eligible for a rollover distribution, EXCEPT:
A. Termination of employment
B. Attainment of age 59½
C. Attainment of NRA
D. Disability
E. Financial hardship
" "The correct answer is E.
This is false because hardship distributions are not eligible for rollover.
"
"All of the following statements regarding a QJSA in a defined benefit plan are TRUE, EXCEPT:
A. A plan may choose to require a participant to be married for at least one year before treating that participant as married for QJSA purposes.
B. A QJSA is a single life annuity for an unmarried participant.
C. The amount of the survivor annuity must not be less than 50% of the amount payable during the period that the participant and spouse are both alive.
D. All married participants are required to receive their distribution in the form of a QJSA.
E. A notice must be provided explaining what a QJSA is.
" "The correct answer is D.
This statement is false because married participants who receive consent from their spouse may elect a different form of payment.
"
"Based on the following information, determine the latest date the participant must begin receiving an RMD:
• The participant’s date of birth is September 1, 1944.
• The participant is a 50% owner.
• The participant is still actively employed.
A. September 1, 2014
B. March 1, 2015
C. April 1, 2015
D. December 31, 2015
E. April 1, 2016
" "The correct answer is E.
A 5% owner is required to take minimum distributions no later than April 1st of the calendar year following the year in which the participant attains age 70½ even if still employed. This participant was born September 1, 1944 and attains age 70½ on March 1, 2015. Therefore, RMDs must begin no later than April 1, 2016.
"
"All of the following statements regarding RMDs are TRUE, EXCEPT:
A. A terminated participant turning 70½ may choose to receive an RMD by December 31 of the year the participant becomes 70½.
B. An RMD is made one time and when made no further distributions need to occur.
C. Participants reaching age 70½ and retiring can postpone receiving their first RMD until April 1st of the calendar year following the year they reach age 70½.
D. Participants who are 5% owners must begin minimum distributions by April 1st of the calendar year following the year in which they attain age 70½ even if still employed.
E. Active participants age 70½, who are not 5% owners, can postpone receiving RMDs until they retire.
" "The correct answer is B.
This statement is false because once begun, an RMD is due each December 31.
"
"All of the following statements regarding QPSAs are TRUE, EXCEPT:
A. The plan may require the spouse to have been married to the participant for 18 months to be eligible for the benefit.
B. The distribution to the spouse must be offered in the form of a QPSA unless the participant and spouse had elected a different form prior to the participant’s death.
C. It is a series of periodic payments for the life of the surviving spouse of a participant who died before commencing payment of benefits.
D. In a money purchase plan the benefit must be equal to an amount of at least 50% of the participant’s vested account balance.
E. The QPSA is determined as if the participant had retired one day prior to the date of death.
" "The correct answer is A.
This statement is false because the plan may require the spouse to have been married to the participant for at least one year to be eligible for the benefit.
"
"All of the following are distributable events for the participant, EXCEPT:
A. Retirement
B. Disability
C. Death
D. QDRO
E. Termination of service
" "The correct answer is D.
This is false because while a QDRO may be a distributable event for the alternate payee, it is not for the participant.
"
"All of the following statements regarding rollovers for a participant who has separated from service are TRUE, EXCEPT:
A. There must be a distributable event for a rollover to occur.
B. There is a 50% excise tax for failure to take the distribution upon termination of employment prior to NRA.
C. The rollover must occur within 60 days of receipt of the distribution.
D. There is a 20% federal income tax withholding on amounts over $200 distributed to the participant and that are eligible rollover distributions.
E. A distribution to a participant who separates from service before age 55 will be subject to a 10% tax on early distributions.
" "The correct answer is B.
This statement is false because there is only a 50% excise tax for failure to take an RMD.
"
"All of the following are exempt from the 10% tax on early distributions, EXCEPT:
A. Amounts rolled over into another plan
B. Distributions due to death
C. Distributions prior to age 59½ for hardship
D. Distributions after age 55 and separation from service
E. In-plan Roth conversion
" "The correct answer is C.
This is false because there is no exception for hardship distribution or plan termination distribution made prior to age 59½.
"
"Which of the following statements regarding distributions is/are TRUE?
I. A distribution from a designated Roth account that occurs when a participant is age 60 and contributions have been in the account for ten years is not taxable.
II. A distribution from a designated Roth account that occurs when a participant is age 30 and contributions have been in the account for two years of service will result in taxable earnings.
III. A distribution from a profit sharing account that occurs when a participant is age 45 and contributions have been in the account for two years of service will result in taxable earnings and contributions.
A. II only
B. III only
C. I and II only
D. I and III only
E. I, II and III
" "The correct answer is E.
All statements are true.
"
"All of the following statements regarding distributions are TRUE, EXCEPT:
A. An in-plan Roth conversion is subject to a 20% federal income tax withholding.
B. An eligible rollover distribution paid directly to the participant is subject to 20% federal income tax withholding.
C. A distribution that is not an eligible rollover distribution paid directly to the participant is subject to a 10% withholding that may be waived by the participant.
D. A distribution amount of $200 or less is not subject to withholding.
E. A distribution that is rolled over directly from the plan to an IRA is not subject to withholding.
" "The correct answer is A.
This statement is false because when an in-plan Roth conversion occurs no withholding applies.
"
"All of the following statements regarding the DOL voluntary correction programs are TRUE, EXCEPT:
A. Delinquent Forms 5500 may be filed under the DOL programs with all or a portion of the civil penalties being waived.
B. Delinquent Forms 1099-R may be filed.
C. Delinquent participant contributions may be corrected.
D. Sale of assets by plans to parties-in-interest may be corrected.
E. Loans issued to parties-in-interest at below market rates may be corrected.
" "The correct answer is B.
This statement is false because the DOL correction programs do not address delinquent Form 1099-Rs.
"
"All of the following statements regarding IRS programs available for correction of plan defects are TRUE, EXCEPT:
A. SCP is available to correct insignificant operational failures at any time.
B. SCP is available to correct failures involving misuse of plan assets.
C. VCP can be used to correct plan document failures.
D. VCP can be used to correct demographic failures.
E. Audit CAP is used to correct qualification defects discovered by the IRS when auditing the plan.
" "The correct answer is B.
This statement is false because SCP is not available to correct failures involving misuse of plan assets.
"
"All of the following describe events that may cause a plan document to be amended, EXCEPT:
A. A change in the IRS determination letter process
B. A new IRS regulation that interprets a law
C. The plan sponsor changes its qualified plan goals
D. A law change that affects the plan
E. A faulty plan provision
" "The correct answer is A.
This is false because a change in the IRS determination letter process does not necessarily require a plan to be amended.
"
"Which of the following statements regarding the potential consequences of plan disqualification is/are TRUE?
I. Earnings on plan assets may become currently taxable to the trust.
II. The employer’s contribution deductions may be disallowed to the extent participants are not vested.
III. Participants may have to include vested benefits in their gross income.
A. I only
B. II only
C. I and III only
D. II and III only
E. I, II and III
" "The correct answer is E.
All statements are true.
"
"All of the following statements regarding amending a qualified plan are TRUE, EXCEPT:
A. The plan document must contain a written procedure for amending the plan.
B. Discretionary amendments must be adopted by the first day of the plan year it is to be effective.
C. Discretionary amendments represent the plan sponsor’s ability to tailor the plan to meet the plan sponsor’s intended purpose.
D. An amendment cannot eliminate benefits or rights that have already accrued.
E. The plan document must state the person or entity that has been granted legal authority to amend the plan.
" "The correct answer is B.
This statement is false because some amendments may be adopted during the plan year they are to be effective, provided protected benefits are not cut back.
"
"All of the following may be corrected under the VFC program, EXCEPT:
A. Delinquent payment of employer contributions
B. Payment of dual compensation to plan fiduciaries
C. Purchase of assets by plan from parties-in-interest
D. Benefit payments based on improper valuation of plan assets
E. Below market interest rate loans with parties-in-interest
" "The correct answer is A.
This is false because delinquent payment of employer contributions cannot be corrected under the VFC.
"
"All of the following may be corrected under EPCRS, EXCEPT:
A. Plan document failures
B. Operational defects
C. Demographic issues
D. Ineligible employer defects
E. Prohibited transactions
" "The correct answer is E.
This is false because prohibited transactions may not be corrected under EPCRS.
"
"Which of the following IRS programs is/are available for qualified plans to correct plan defects?
I. VCP
II. DFVC
III. SCP
A. I only
B. II only
C. I and III only
D. II and III only
E. I, II and III
" "The correct answer is C.
Item II is false since the DFVC is a DOL program used for delinquent Form 5500 filings.
"
"Which of the following statements regarding the potential consequences of plan disqualification is/are TRUE?
I. Participant loans may have to be repaid.
II. Participants may have to include vested benefits in their gross income.
III. Tax-free rollovers may be disallowed.
A. I only
B. II only
C. I and III only
D. II and III only
E. I, II and III
" "The correct answer is D.
Statement I is false because participant loans would not have to be repaid.
"
"All of the following are events that may cause a plan document to be amended EXCEPT:
A. Flawed language
B. A change in marital status
C. An expanded employee population
D. Adoption of a loan program
E. Change in corporate structure
" "The correct answer is B.
This is false because a change in marital status may cause a participant to change its beneficiary but it would not cause an amendment to the plan.
"
"All of the following plan records should be retained by the plan sponsor, EXCEPT:
A. Full and complete plan document properly dated and signed
B. Form 5500
C. Letters of appreciation to participants for jobs well done
D. SPD
E. Evidence of the plan’s fidelity bond
" "The correct answer is C.
This is false because letters of appreciation to participants for jobs well done need not be retained for qualified plan purposes.
"
"All of the following standards must be met by a recordkeeping system utilizin the safe harbor method for storage of records on electronic media, EXCEPT:
A. Have reasonable controls to ensure accuracy.
B. Be convertible to paper copies.
C. Be capable of retrieving the information.
D. Be capable of indexing.
E. Be kept in a secure room monitored by video cameras.
" "The correct answer is E.
This is false because there is no requirement that the recordkeeping system be kept in a secure room monitored by video cameras.
"
"Which of the following is/are employee data records that need to be kept?
I. Those records that help determine the benefits due
II. Just those records for participating employees in a 401(k) plan
III. Records for all employees, not just those eligible, in a profit sharing plan
A. I only
B. II only
C. I and III only
D. II and III only
E. I, II and III
" "The correct answer is C.
Statement II is false since all employee records must be kept to show how eligibility was determined.
"
"All of the following should be documented and archived each year, EXCEPT:
A. Sales meeting minutes
B. QDRO procedures
C. Key employee information
D. SPD
E. Controlled group information
" "The correct answer is A.
This is false because sales meeting minutes do not need documented and archived for qualified retirement plan purposes.
"
"All of the following statements regarding safe harbor electronic document record retention requirements are TRUE, EXCEPT:
A. The recordkeeping system must be capable of reproducing the electronic record.
B. The recordkeeping system must allow for ready access of the records at reasonable times.
C. The recordkeeping system must be capable of preserving the electronic records.
D. A designated time and method to archive must be in place.
E. Standard records management practices are established and implemented.
" "The correct answer is D.
This statement is false because a designated time and method to archive does not need to be in place.
"
"All of the following documents should be kept indefinitely, EXCEPT:
A. Employee’s salary history
B. Employee’s hours history
C. Plan financial reports
D. Plan document
E. Amount of benefits paid
" "The correct answer is C.
This is false because plan financial reports do not have to be kept indefinitely.
"
"All of the following are safe harbor electronic document record management practices and audits requirements, EXCEPT:
A. Selecting an off-site storage location
B. Providing an indestructible storage facility
C. Providing a secure storage environment
D. Observing a quality assurance program
E. Labeling of records
" "The correct answer is B.
This is false because providing an indestructible storage facility is not a safe harbor records management practice.
"
"Which of the following is/are paper records that should be kept indefinitely even though they are transferred to an electronic recordkeeping system?
I. Stock certificates
II. Documents executed under seal
III. Notarized documents
A. I only
B. III only
C. I and II only
D. II and III only
E. I, II and III
" "The correct answer is E.
All are true.
"
"Which of the following is/are employee data records that must be kept?
I. Information needed to determine vested terminated participant’s amount of benefits
II. Information needed to determine vested terminated participant’s entitlement to a benefit
III. Information needed to determine active participant’s amount of benefits
A. II only
B. III only
C. I and II only
D. II and III only
E. I, II and III
" "The correct answer is E.
All are true.
"
"All of the following are safe harbor electronic media storage requirements EXCEPT:
A. The system provides that after documents are electronically stored all paper copies are maintained for another 3 years
B. The system is capable of retaining the records
C. The system may convert the electronic documents into legible paper copies
D. The system does not require participant to sign on at a particular site in order to access the records
E. The system is not subject to restriction that would limit access to the records
" "The correct answer is A.
This is false because after the documents are copied electronically most of them can be immediately disposed of unless they have legal significance, an inherent value, are not clear on the electronic media or not legible when reproduced on paper.